Cts approximation, delta function integration, stat mech

Click For Summary
The discussion centers on the use of the continuous approximation in delta function integration within statistical mechanics. It highlights that the condition for the approximation holds when E divided by ħω is significantly greater than one. The integration of the delta function leads to a distinction based on whether the argument falls within the integration limits, specifically relating to the energy E compared to ħω/2. The confusion arises regarding how the delta function's properties translate into inequalities for E. Ultimately, the key takeaway is that the delta function evaluates to zero or one based on the value of E relative to ħω/2.
binbagsss
Messages
1,291
Reaction score
12

Homework Statement



g(E) question.png


Homework Equations

The Attempt at a Solution



So cts approx holds because ##\frac{E}{\bar{h}\omega}>>1##

So
##\sum\limits^{\infty}_{n=0}\delta(E-(n+1/2)\bar{h} \omega) \approx \int\limits^{\infty}_{0} dx \delta(E-(x+1/2)\bar{h}\omega) ##

Now if I do a substitution ##x'=x\bar{h}\omega## to loose the ##\bar{h}\omega## multiplying the ##x## , ##dx'=\bar{h}\omega dx##

I get
## dx' \frac{1}{\bar{h}\omega}\int\limits^{\infty}_{0}\delta(x'-(E-\frac{1}{2}\bar{h}\omega)) ##

Now, if I denote the region that ##x'## is integrated over by ##D## I get that this is:
##= \frac{1}{\bar{h}\omega} ## if ##x'=E-1/2\bar{h}\omega \in D##
##= 0 ## if ##x'=E-1/2\bar{h}\omega \notin D##

The solution however has:

gE sol.png


##= \frac{1}{\bar{h}\omega} ## if ##E>1/2\bar{h}\omega ##
##= 0 ## if ##E<1/2\bar{h}\omega ##

Excuse me if I'm being stupid but I have no idea how we have converted the requirements of a certain value of ##x'## to lie inside the region of integration or not, which I believe is the definition of the delta function, to inequalities imposed on ##E##?


Many thanks in advance
 
Physics news on Phys.org
binbagsss said:
## dx' \frac{1}{\bar{h}\omega}\int\limits^{\infty}_{0}\delta(x'-(E-\frac{1}{2}\bar{h}\omega)) ##

Don't you mean for dx&#039; to be inside the integral? If so, you're basically done. For any x_0,

\int_0^{\infty} \delta(x&#039; - x_0) dx&#039; = 0 if x_0 &lt; 0
\int_0^{\infty} \delta(x&#039; - x_0) dx&#039; = 1 if x_0 &gt;0

So we have the particular case x_0 = E-\frac{1}{2}\bar{h}\omega
 
  • Like
Likes binbagsss
Question: A clock's minute hand has length 4 and its hour hand has length 3. What is the distance between the tips at the moment when it is increasing most rapidly?(Putnam Exam Question) Answer: Making assumption that both the hands moves at constant angular velocities, the answer is ## \sqrt{7} .## But don't you think this assumption is somewhat doubtful and wrong?

Similar threads

Replies
1
Views
2K
  • · Replies 3 ·
Replies
3
Views
3K
  • · Replies 3 ·
Replies
3
Views
2K
  • · Replies 3 ·
Replies
3
Views
2K
  • · Replies 7 ·
Replies
7
Views
2K
Replies
2
Views
2K
  • · Replies 2 ·
Replies
2
Views
2K
  • · Replies 2 ·
Replies
2
Views
2K
Replies
2
Views
2K
  • · Replies 3 ·
Replies
3
Views
1K